- PowerScore Staff
- Posts: 5972
- Joined: Mar 25, 2011
- Thu Jan 21, 2016 12:00 am
#72531
Complete Question Explanation
Weaken-SN. The correct answer choice is (D)
This is another difficult problem but very typical of the LSAT. As always, the key to success is to isolate the conclusion, which appears in the last sentence: “companies producing carpet will be able to gain market share in the carpet market only through purchasing competitors.” As you should have noted while reading, the conclusion contains a conditional indicator and is thereby conditional in nature. The conclusion can be diagrammed as:
Answer choice (A): This answer goes beyond the scope of the argument, which is limited to the carpet market (and not other floor coverings).
Answer choice (B): This is an Opposite answer that strengthens the argument. If there are no remaining niches to fill, then there is no way to expand other than to purchase a competitor.
Answer choice (C): This attractive answer is wrong for two very strong reasons:
Answer choice (D): This is the correct answer. If price reductions drive out some of the carpet producers, then other producers can take the market share left behind. This scenario shows that a company can gain market share without purchasing a competitor, thus attacking the necessary condition in the conclusion.
Answer choice (E): This Opposite answer strengthens the argument. If the consumers are resistant to new styles, then one fewer possibility exists if a company is trying to increase market share. By eliminating this option, the conclusion is strengthened (if you eliminate an idea that would hurt the argument, that strengthens the argument because it has fewer “competitors”).
Weaken-SN. The correct answer choice is (D)
This is another difficult problem but very typical of the LSAT. As always, the key to success is to isolate the conclusion, which appears in the last sentence: “companies producing carpet will be able to gain market share in the carpet market only through purchasing competitors.” As you should have noted while reading, the conclusion contains a conditional indicator and is thereby conditional in nature. The conclusion can be diagrammed as:
- GMS = gain market share in the carpet market
PC = purchasing competitors
GMS PC
Answer choice (A): This answer goes beyond the scope of the argument, which is limited to the carpet market (and not other floor coverings).
Answer choice (B): This is an Opposite answer that strengthens the argument. If there are no remaining niches to fill, then there is no way to expand other than to purchase a competitor.
Answer choice (C): This attractive answer is wrong for two very strong reasons:
- 1. A Shell Game is played with the details of the conclusion. The conclusion is about market share. Answer choice (C) is about a decline in profits and revenues. The two are not the same, and so the information in the answer choice does not weaken the conclusion.
2. Even if you assume that market share is the same thing as profits and revenues, a second Shell Game is played because the answer then attacks a conclusion that is similar but different from the given conclusion.
If the conclusion were as follows:
PC GMS
Then answer choice (C) would be correct (again, assuming market share is the same thing as profits and revenues). But, the above representation is a Mistaken Reversal of the conclusion, and so the attack is made on a statement that uses the same terms as the conclusion but puts them in a different relationship. Stop for a moment and think about how this works, because this is a great example of the cleverness displayed by the test makers. They’ve presented an answer that would effectively attack a conclusion that is very similar to the actual conclusion in the argument (but that is in fact different). Fortunately you can avoid this answer if you know what to look for when attacking conditional reasoning.
Answer choice (D): This is the correct answer. If price reductions drive out some of the carpet producers, then other producers can take the market share left behind. This scenario shows that a company can gain market share without purchasing a competitor, thus attacking the necessary condition in the conclusion.
Answer choice (E): This Opposite answer strengthens the argument. If the consumers are resistant to new styles, then one fewer possibility exists if a company is trying to increase market share. By eliminating this option, the conclusion is strengthened (if you eliminate an idea that would hurt the argument, that strengthens the argument because it has fewer “competitors”).
Dave Killoran
PowerScore Test Preparation
Follow me on X/Twitter at http://twitter.com/DaveKilloran
My LSAT Articles: http://blog.powerscore.com/lsat/author/dave-killoran
PowerScore Podcast: http://www.powerscore.com/lsat/podcast/
PowerScore Test Preparation
Follow me on X/Twitter at http://twitter.com/DaveKilloran
My LSAT Articles: http://blog.powerscore.com/lsat/author/dave-killoran
PowerScore Podcast: http://www.powerscore.com/lsat/podcast/